Välkommen till Resurscentrums frågelåda!

 

Vill du ha ett snabbt svar - sök i databasen: Anpassad Google-sökning
(tips för sökningen).
Använd diskussionsforum om du vill diskutera något.
Senaste frågorna. Veckans fråga.

2 frågor/svar hittade

Kraft-Rörelse [15982]

Fråga:
Gausspistol
/Veckans fråga

Ursprunglig fråga:
Hur fungerar "Gauss pistol", förenklad variant - man har en ränna med en magnet och några stålkulor på rad efter, sedan rullar man en ytterligare kula mot magneten varefter den sista kulan i ledet skjuts i väg med en mycket högre fart.

(Finns på youtube under länk 1 - jag har testat och det fungerar även med en mindre vanlig magnet.)
/Anton O, Myrviken, Myrviken

Svar:
Anton! Kul leksak! Jag har inte sett den tidigare!

Apparaten (se videon nedan) består av en ränna, ett par starka magneter och några stålkulor.

Funktionen är lite relaterad till Newtons vagga, se fråga 11464 . Den fysikaliska grunden är, förutom magnetism, lagarna om energins och rörelsemängdens bevarande.

Skillnaden jämfört med Newtons vagga är att den högra kulan (vi refererar nu till bilden nedan) kommer att accelereras mot magnet/stålkule paketet av magneten. Detta innebär att rörelsemängd överföres från den rörliga kulan till det från början stationära paketet kulor/magneter medan kulan rör sig från höger till vänster. I fallet Newtons vagga accelereras den fria kulan av tyngdkraften och ger alltså ingen rörelemängdsöverföring till de stationära kulorna före kollisionen.

Magneten behövs alltså för att sätta fart på den högra kulan. Den behövs även för att hålla ihop paketet av kulorna till vänster. Dessa behövs för att ge en stor massa på den från början stationära biten. Ett ytterligare skäl för att ha mer än en stationär kula är att den vänstraste kulan (som flyger iväg) inte får sitta alltför hårt fast. Om den gjorde det skulle den inte kunna flyga iväg.

Detta är vad som sker:

1 Den högra kulan rör sig långsamt mot magneterna.

2 När kulan närmar sig magneterna accelereras den mer och mer mot magneterna. För att bevara rörelsemängden kommer de stationära kulorna och magneterna att röra sig lite åt höger.

3 Den högra kulan kolliderar med magneten. Stöten fortplantas genom magneterna och de stationära kulorna.

4 Av samma skäl som för Newtons vagga blir det kulan längst till vänster som tar upp den rörelsemängd som den högra kulan hade vid kollisionen.

5 Lägg märke till att "restpaketet" rör sig lite åt höger efter stöten (till skillnad från Newtons vagga). Anledningen är att kraften som accelererade den högra kulan kom från det från början stationära paketet.

"Gauss gun" kan även vara en pistol med ferromagnetisk kula som accelereras med hjälp av varierande magnetfält i ett antal spolar, se Coilgun . Detta är alltså någonting helt annat och mycket farligare än vad som beskrivs ovan.

Anledningen till benämningen "Gauss pistol" är naturligtvis att magnetfält är involverade. Se även länk 1.


/Peter E

Nyckelord: Newtons vagga [2]; elastisk stöt [12]; magnetism [52];

1 http://scitoys.com/scitoys/scitoys/magnets/gauss.html

*

Kraft-Rörelse [11464]

Fråga:
Newtons vagga
/Veckans fråga

Ursprunglig fråga:
Funderingar kring det klassiska experimentet, då man har ett antal intilliggande kulor i paralella snören. Man låter en av kulorna stöta till hela raden varpå den yttersta far iväg med samma fart som den som slog i hela raden. Varför är det bara den yttersta som far iväg? Skulle man inte kunna behålla rörelsemängden om man lät t.ex. de övriga fyra kulorna fara iväg med en lägre fart? Fast då verkar inte den kinetiska energin bevaras...? Är det så att det är den enda möjliga kulkombinationen för en perfekt elastisk stöt att bevara både energi och rörelsemängd?
/Mattias N, Hvitfeldska, Göteborg

Svar:
Denna anordning brukar kallas Newtons vagga, se bilden nedan från Wikimedia Commons (Newton's_cradle ).

Du är inne på helt rätt spår! Både energi och rörelsemängd måste bevaras. Detta om vi förutsätter helt elastiska kollisioner, vilket ju inte är helt uppfyllt. Kulorna stannar ju såsmåningom.

Antag att varje kula väger m gram, vi lyfter n kulor och dessa har hastigheten v när de kolliderar med de stationära kulorna. Vi måste bevara rörelsemängden n*m*v och rörelseenergin n*m*v2/2. Med dessa antaganden måste både n och v bevaras.

Denna förenklade förklaring tillfredsstäler nog de flesta, men om du inte är nöjd finns en mycket ingående diskussion i Newton's Cradle .

Länk 1 innehåller en java-applikation med vaggan. Se även länk 2 under "Exercises and answers".



/Peter E

Nyckelord: Newtons vagga [2]; elastisk stöt [12];

1 http://www.sciencejoywagon.com/physicszone/lesson/otherpub/wfendt/newtonscradle.htm
2 http://2014.plancks.info/

*

Ämnesområde
Sök efter
Grundskolan eller gymnasiet?
Nyckelord: (Enda villkor)
Definition: (Enda villkor)
 
 

Om du inte hittar svaret i databasen eller i

Sök i svenska Wikipedia:

- fråga gärna här.

 

 

Frågelådan innehåller 7624 frågor med svar.
Senaste ändringen i databasen gjordes 2022-05-21 17:33:39.

 

** Frågelådan är stängd för nya frågor tills vidare **


sök | söktips | Veckans fråga | alla 'Veckans fråga' | ämnen | dokumentation | ställ en fråga
till diskussionsfora

 

Creative Commons License

Denna sida från NRCF är licensierad under Creative Commons:
Erkännande-Ickekommersiell-Inga bearbetningar
.